How Do You Find Points on a Graph Where the Tangent Line Has a Specific Slope?

  • Thread starter Thread starter brettevan
  • Start date Start date
  • Tags Tags
    Graph Points
Click For Summary
To find points on the graph of the function y=1/3x^3-2x^2+8x+14 where the tangent line has a slope of 5, one must calculate the derivative of the function. The derivative represents the slope of the tangent line at any point on the graph. Clarification on the function's notation is necessary, as there are multiple interpretations of the equation presented. Properly defining the function will aid in solving for the points where the derivative equals 5. Understanding derivatives is essential for addressing the problem effectively.
brettevan
Messages
1
Reaction score
0
I have absolutely no idea how to solve this problem and I would really appreciate same help.

For the function, find the point(s) on the graph a which the tangent line has slope 5

y=1/3x^3-2x^2+8x+14

Any help with this would br greatly appreciated
 
Physics news on Phys.org
Do you know what a derivative is and what it might have to do with a slope? You really have to make a better start than that.
 
brettevan said:
I have absolutely no idea how to solve this problem and I would really appreciate same help.

For the function, find the point(s) on the graph a which the tangent line has slope 5

y=1/3x^3-2x^2+8x+14

Any help with this would br greatly appreciated

Use brackets. I really cannot figure out whether you mean y = \frac{1}{3} x^3 - 2 x^2 + 8x + 14 or \frac{1}{3x^3} - 2x^2 + 8x + 14 or y =\frac{1}{3x^3 - 2x^2 + 8x + 14} . Strictly according to standard rules, what you wrote is the first of these three possibilities. If you don't want to use Latex you can just write (1/3)x^3 - 2x^2 + ... or 1/(3x^3) - 2x^2 + ... or 1/(3x^3 - 2x^2 + ...)

RGV
 
Question: A clock's minute hand has length 4 and its hour hand has length 3. What is the distance between the tips at the moment when it is increasing most rapidly?(Putnam Exam Question) Answer: Making assumption that both the hands moves at constant angular velocities, the answer is ## \sqrt{7} .## But don't you think this assumption is somewhat doubtful and wrong?

Similar threads

Replies
2
Views
1K
  • · Replies 2 ·
Replies
2
Views
2K
  • · Replies 11 ·
Replies
11
Views
2K
Replies
4
Views
1K
  • · Replies 11 ·
Replies
11
Views
2K
  • · Replies 6 ·
Replies
6
Views
2K
  • · Replies 2 ·
Replies
2
Views
2K
  • · Replies 6 ·
Replies
6
Views
2K
  • · Replies 1 ·
Replies
1
Views
1K
Replies
5
Views
2K